Đến nội dung

Hình ảnh

Tổng hợp các bài BĐT

ẩn đi không cho xem

  • Please log in to reply
Chủ đề này có 164 trả lời

#141
an1712

an1712

    Trung sĩ

  • Thành viên
  • 149 Bài viết

$P=2(\frac{x^5}{x}+\frac{x^5}{x})+2x^8-4(1+x^2)^2$\

$\rightarrow P=4x^4+2x^8-4(1+x^2)^2$

Từ đây tìm ra x,y để định hướng cho bài toán


tiến tới thành công  :D


#142
DOTOANNANG

DOTOANNANG

    Đại úy

  • ĐHV Toán Cao cấp
  • 1609 Bài viết

C h ứ n g  m i n h  v ớ i  $\left \vert \it{c} \right \vert\leqq \it{b}$  $:$

$$\begin{equation}\begin{split} \left ( \frac{21\,a^{\,3}- 17\,ab^{\,2}- 4\,c^{\,3}}{21} \right )^{\,2}- (\,a^{\,2}- b^{\,2}\,)^{\,3}\geqq 0 \end{split}\end{equation}$$

$<$$=$$>$

$$\begin{equation}\begin{split} \text{leftside}=  \end{split}\end{equation}$$

$$\begin{equation}\begin{split} \left ( \frac{21\,a^{\,3}- 17\,ab^{\,2}- 4\,c^{\,3}}{21} \right )^{\,2}+ (\,b^{\,2}- a^{\,2}\,)^{\,3} \end{split}\end{equation}$$

$$\begin{equation}\begin{split} = \left ( \frac{4\,b^{\,2}+ 4\,c^{\,2}}{21} \right )^{\,2}+ \left [ \frac{(\,a+ b\,)b}{21} \right ]^{\,2}(\,609\,a^{\,2}- 1050\,ab+ 457\,b^{\,2}\,)+ \frac{2(\,4\,b^{\,3}+ 4\,c^{\,3}\,)(\,-\,a- b\,)(\,4\,b^{\,2}- 21\,ab+ 21\,a^{\,2})}{21^{\,2}} \end{split}\end{equation}$$

$$\begin{equation}\begin{split} = \left ( \frac{4\,b^{\,2}- 4\,c^{\,2}}{21} \right )^{\,2}+ \left [ \frac{(\,a- b\,)b}{21} \right ]^{\,2}(\,609\,a^{\,2}+ 1050\,ab+ 457\,b^{\,2}\,)+ \frac{2(\,4\,b^{\,3}- 4\,c^{\,3}\,)(\,a- b\,)(\,4\,b^{\,2}+ 21\,ab+ 21\,a^{\,2})}{21^{\,2}} \end{split}\end{equation}$$



#143
nguyen minh hieu hp

nguyen minh hieu hp

    Binh nhất

  • Thành viên mới
  • 25 Bài viết

Cho a,b,c>0 và $\sum a^{2}=1$

Chứng minh rằng: $\sum \frac{1}{\sqrt{a^{2}+1}}\leq \frac{9}{2(a+b+c)}$



#144
Gammaths11

Gammaths11

    Hạ sĩ

  • Thành viên
  • 53 Bài viết

áp dung bđt bunhia: $VT^{2}\leq 3\left (\sum \frac{1}{a^{2}+1} \right )\Leftrightarrow \frac{VT^{2}}{3}\leq 3-\sum \frac{a^{2}}{a^{2}+1}\leq 3-\frac{(a+b+c)^{2}}{a^{2}+b^{2}+c^{2}+3}= 3-\frac{(a+b+c)^{2}}{4}$

ta phải chứng minh $\frac{VT^{2}}{3}\leq \frac{VP^{2}}{3}$

$\Leftrightarrow 3-\frac{(a+b+c)^{2}}{4}\leq \frac{27}{4(a+b+c)^{2}} \Leftrightarrow \frac{27}{4(a+b+c)^{2}}+\frac{(a+b+c)^{2}}{4}\geq 3$

Ta có :$[\frac{9}{4(a+b+c)^{2}}+\frac{(a+b+c)^{2}}{4} ]+\frac{9}{2(a+b+c)^{2}}\geq \frac{3}{2}+\frac{9}{2.3(a^{2}+b^{2}+c^{2})}= \frac{3}{2}+\frac{3}{2}=3$



#145
Gallan

Gallan

    Lính mới

  • Thành viên mới
  • 3 Bài viết

<3


Bài viết đã được chỉnh sửa nội dung bởi Gallan: 05-08-2021 - 17:09


#146
Pray for The First

Pray for The First

    Lính mới

  • Thành viên mới
  • 5 Bài viết
Giúp em bài này với ạ

Hình gửi kèm

  • IMG_20220926_183918.jpg


#147
Le Tuan Canhh

Le Tuan Canhh

    Thượng sĩ

  • Thành viên
  • 226 Bài viết

Bài của Pray for The First

$P=\frac{(a^{2}+1)(b+c)+a^{2}+(b+c)^{2}+4}{a+b+c}\geq \frac{2a(b+c)+a^{2}+(b+c)^{2}+4}{a+b+c}=\frac{(a+b+c)^{2}+4}{a+b+c}\geq 4$

Dấu "=" xảy ra khi $a=1;b+c=1$

 

 


Dư :unsure: Hấu   


#148
ThienDuc1101

ThienDuc1101

    Hạ sĩ

  • Thành viên
  • 59 Bài viết

Giúp em bài này với ạ.

Cho $x,y,z$ là các số thực dương thỏa mãn $x+y+z=3$. Tìm GTNN của $P=\frac{x+y}{\sqrt{x^2+y^2+6z}}+\frac{y+z}{\sqrt{y^2+z^2+6x}}+\frac{z+x}{\sqrt{z^2+x^2+6y}}$


Bài viết đã được chỉnh sửa nội dung bởi ThienDuc1101: 27-09-2022 - 19:48


#149
thanhng2k7

thanhng2k7

    Trung sĩ

  • Thành viên
  • 129 Bài viết

Giúp em bài này với ạ.

Cho $x,y,z$ là các số thực dương thỏa mãn $x+y+z=3$. Tìm GTNN của $P=\frac{x+y}{\sqrt{x^2+y^2+6z}}+\frac{y+z}{\sqrt{y^2+z^2+6x}}+\frac{z+x}{\sqrt{z^2+x^2+6y}}$

Với $x+y+z=3$ thay vào P ta được :

 $P= \frac{3-z}{\sqrt{x^2+y^2+6(3-x-y)}}+\frac{3-x}{\sqrt{z^2+y^2+6(3-z-y)}}+\frac{3-y}{\sqrt{x^2+z^2+6(3-x-z)}}$

$\Rightarrow P = \sum_{x,y,z}{\frac{(z-3)^2}{(x-3)^2+(y-3)^2}}$

Đặt ẩn phụ như sau:

$(x-3)^2=a;(y-3)^2=b;(z-3)^2=c \Rightarrow a,b,c\geq 0$

$\Rightarrow P=\sum_{a,b,c}\sqrt{\frac{a}{b+c}}$

Sử dụng AM-GM dưới dạng mẫu :

$\sum_{a,b,c}\sqrt{\frac{a}{b+c}}=\sum_{a,b,c}\frac{2a}{2\sqrt{a(b+c)}}\geq \sum_{a,b,c}\frac{2a}{a+b+c}= 2$

Dấu "=" xảy ra khi $(x,y,z)=(3,0,0)$ và các hoán vị . 


Tất cả mọi thứ đều có thể chứng minh bằng Toán học ;)


#150
Matthew James

Matthew James

    Trung sĩ

  • Thành viên
  • 106 Bài viết

Với ba số $x,y,z$ thỏa mãn điều kiện $x+y+z=1$, chứng minh rằng:

$\frac{1-x^2}{x+yz}+\frac{1-y^2}{y+zx}+\frac{1-z^2}{z+xy}\geq 6$


Mathematics reveals its secrets only to those who approach it with pure love, for its own beauty. :D 


#151
Le Tuan Canhh

Le Tuan Canhh

    Thượng sĩ

  • Thành viên
  • 226 Bài viết

Xét riêng: $\frac{1-x^{2}}{x+yz}=\frac{(x+y+z)^{2}-x^{2}}{x(x+y+z)+yz}=\frac{(2x+y+z)(y+z)}{(x+y)(x+z)}=\frac{y+z}{x+y}+\frac{y+z}{x+z}$

Tương tự có : $\frac{1-y^{2}}{y+zx}=\frac{z+x}{y+x}+\frac{z+x}{y+z}$

                       $\frac{1-z^{2}}{z+xy}=\frac{x+y}{z+x}+\frac{x+y}{z+y}$

Chịu khó quan sát và cộng hết lại ta có :

           $VT=\frac{x+2y+z}{x+z}+\frac{x+y+2z}{x+y}+\frac{2x+y+z}{y +z}=3+2(\frac{y}{x+z}+\frac{z}{x+y}+\frac{x}{y+z})\geq 6$


Dư :unsure: Hấu   


#152
Matthew James

Matthew James

    Trung sĩ

  • Thành viên
  • 106 Bài viết

3+2(\frac{y}{x+z}+\frac{z}{x+y}+\frac{x}{y+z})\geq 6$

 

Anh ơi đoạn này em chứng minh dùng bđt Cauchy-Swcharz thì nó không ra ạ:

$(\frac{y}{x+z}+\frac{z}{x+y}+\frac{x}{y+z})\geq \frac{(x+y+z)^2}{2(xy+yz+xz)}\geq \frac{1}{(x+y+z)^2}=1$


Mathematics reveals its secrets only to those who approach it with pure love, for its own beauty. :D 


#153
Matthew James

Matthew James

    Trung sĩ

  • Thành viên
  • 106 Bài viết

 

3+2(\frac{y}{x+z}+\frac{z}{x+y}+\frac{x}{y+z})\geq 6$

 

Anh ơi đoạn này em chứng minh dùng bđt Cauchy-Swcharz thì nó không ra ạ:

$(\frac{y}{x+z}+\frac{z}{x+y}+\frac{x}{y+z})\geq \frac{(x+y+z)^2}{2(xy+yz+xz)}\geq \frac{1}{(x+y+z)^2}=1$

 

À thôi em ra rồi.

$(\frac{y}{x+z}+\frac{z}{x+y}+\frac{x}{y+z})\geq \frac{(x+y+z)^2}{2(xy+yz+xz)}\geq \frac{3(xy+yz+zx)}{2(xy+yz+xz)}$


Mathematics reveals its secrets only to those who approach it with pure love, for its own beauty. :D 


#154
ThienDuc1101

ThienDuc1101

    Hạ sĩ

  • Thành viên
  • 59 Bài viết

 

3+2(\frac{y}{x+z}+\frac{z}{x+y}+\frac{x}{y+z})\geq 6$

 

Anh ơi đoạn này em chứng minh dùng bđt Cauchy-Swcharz thì nó không ra ạ:

$(\frac{y}{x+z}+\frac{z}{x+y}+\frac{x}{y+z})\geq \frac{(x+y+z)^2}{2(xy+yz+xz)}\geq \frac{1}{(x+y+z)^2}=1$

 

Chỗ này em nghĩ nếu dùng Cauchy - Schwarz thì nó sẽ không chặt. Thay vào đó dùng BĐT Nesbit sẽ nhanh và chính xác hơn anh ạ.



#155
Matthew James

Matthew James

    Trung sĩ

  • Thành viên
  • 106 Bài viết

Cho $x,y,z$ là các số không âm. CMR:

$4(xy+yz+xz)\leq \sqrt{(x+y)(y+z)(z+x)}(\sqrt{x+y}+\sqrt{y+z}+\sqrt{z+x})$


Mathematics reveals its secrets only to those who approach it with pure love, for its own beauty. :D 


#156
Matthew James

Matthew James

    Trung sĩ

  • Thành viên
  • 106 Bài viết

Cho $a,b,c>0$ thỏa mãn $a+2b+3c \geq 10$, Chứng minh rằng: $a+b+c+\frac{3}{4a}+\frac{9}{8b}+\frac{1}{c} \geq \frac{13}{2}$


Bài viết đã được chỉnh sửa nội dung bởi Matthew James: 05-10-2022 - 20:06

Mathematics reveals its secrets only to those who approach it with pure love, for its own beauty. :D 


#157
Matthew James

Matthew James

    Trung sĩ

  • Thành viên
  • 106 Bài viết

Cho $a,b,c>0$ thỏa mãn $a+2b+3c \geq 10$, Chứng minh rằng: $a+b+c+\frac{3}{4a}+\frac{9}{8b}+\frac{1}{c} \geq \frac{13}{2}$

 

Hình gửi kèm

  • Screenshot 2022-10-05 213338.png

Bài viết đã được chỉnh sửa nội dung bởi Matthew James: 05-10-2022 - 22:06

Mathematics reveals its secrets only to those who approach it with pure love, for its own beauty. :D 


#158
ThienDuc1101

ThienDuc1101

    Hạ sĩ

  • Thành viên
  • 59 Bài viết

Giúp em câu này với ạ.

Cho $a,b,c$ là các số thực dương thỏa mãn $ab+bc+ca=3$. Chứng minh

$\frac{(1+b)^2(1+c)^2}{1+a^2}+\frac{(1+c)^2(1+a)^2}{1+b^2}+\frac{(1+a)^2(1+b)^2}{1+c^2}\geq 24$



#159
duy030408

duy030408

    Binh nhì

  • Thành viên mới
  • 15 Bài viết

Giúp em câu này với ạ.

Cho $a,b,c$ là các số thực dương thỏa mãn $ab+bc+ca=3$. Chứng minh

$\frac{(1+b)^2(1+c)^2}{1+a^2} + \frac{(1+b)^2(1+c)^2}{1+a^2} + \frac{(1+a)^2(1+b)^2}{1+c^2} \geq \frac{16bc}{1+a^2} + \frac{16ac}{1+b^2} + \frac{16ab}{1+c^2}$

Ta có BĐT : $(b+1)^2\geq 4b$ , $(c+1)^2\geq 4c$ , $(a+1)^2\geq 4a$

Suy ra: $\frac{(1+b)^2(1+c)^2}{1+a^2} + \frac{(1+b)^2(1+c)^2}{1+a^2} + \frac{(1+a)^2(1+b)^2}{1+c^2} \geq \frac{16bc}{1+a^2} + \frac{16ac}{1+b^2} + \frac{16ab}{1+c^2}$

Suy ra : $\frac{16c^2b^2}{bc+a^2bc} + \frac{16a^2c^2}{ac+ab^2c} + \frac{16a^2b^2}{ab+abc^2} \geq \frac{16(ab+ac+bc)^2}{ab+ac+bc+abc(a+b+c)}$

Mà ta có : $ab+ac+bc=\frac{(ab+ac+bc)^2}{3}$

                 $abc(a+b+c)\leq \frac{(ab+ac+bc)^2}{3}$

Suy ra:$\frac{16(ab+ac+bc)^2}{ab+ac+bc+abc(a+b+c)}\geq \frac{16(ab+ac+bc)^2}{\frac{2}{3}(ab+ac+bc)^2} = 24$


Bài viết đã được chỉnh sửa nội dung bởi duy030408: 23-04-2023 - 12:01


#160
Lee Ziao Jun

Lee Ziao Jun

    Lính mới

  • Thành viên mới
  • 2 Bài viết

 

Bài 61:

Chứng minh: $(1+\frac{4a}{b+c})$$(1+\frac{4b}{a+c})$$(1+\frac{4c}{a+b})$ $>$25

Biết $a,b,c>0$

Giải:

Vì vai trò của $a,b,c$ là như nhau cho nên ta có thể giả sử rằng: $0<a\leq b\leq c$

 

Đặt $S=a+b+c$, bất đẳng thức cần chứng minh tương đuơng với:

 

$(S+3a)(S+3b)(S+3c)>25S(S-a)(S-b)(S-c)$

 

$\Leftrightarrow S^{3}+2S^{2}(a+b+c)+9S(ab+bc+ca)+27abc>25S^{3}-25S^{2}(a+b+c)+25S(ab+bc+ca)-25abc$

 

$\Leftrightarrow 4S^{3}-16S(ab+bc+ca)+52abc>0$

 

$\Leftrightarrow S(S^{2}-4(ab+bc+ca))+13abc>0$

 

$\Leftrightarrow S((a+b-c)^{2}-4ab)+13abc>0$

 

$\Leftrightarrow S(a+b-c)^{2}+ab(13c-4S)>0$

 

Bất đẳng thức cuối cùng đúng vì $13c-4S=13c-4(a+b+c)=9c-4(a+b)>0$

 

 

 

Bài 62:

Cho $a,b,c >0$ $\sum \frac{(2a+b+c)^2}{2a^2+(b+c)^2}\leq 8$

Giải:

Do bất đẳng thức đã cho là thuần nhất nên ta có thể chuẩn hóa $a+b+c=3$

Do đó bất đẳng thức trở thành 

                     $\sum \frac{(a+3)^2}{2a^2+(3-a)^2} \leq 8\Leftrightarrow \sum \frac{a^2+6a+9}{a^2-2a+3} \leq 24$

Đến đây ta dùng phương pháp hệ số bất định như sau : 

Giả sử $\frac{a^2+6a+9}{a^2-2a+3} \leq k(a-1)+8$

 $\Leftrightarrow ka^3+(7-3k)a^2+(5k-22)a+15-3k \geq 0$

Ta phải viết $ka^3+(7-3k)a^2+(5k-22)a+15-3k=(a-1)^2(ka+15-3k)$

Khai triển, đồng nhất hệ số ta được $k=4$

Vậy ta có $\frac{a^2+6a+9}{a^2-2a+3} \leq 4(a-1)+8$ với mọi $a>0$

Tương tự ta cũng có $\frac{b^2+6b+9}{b^2-2b+3} \leq 4(b-1)+8$

                                  $\frac{c^2+6c+9}{c^2-2c+3} \leq 4(c-1)+8$

Cộng 3 bất đẳng thức trên lại và để ý $a+b+c=3$, ta có được

                 $\sum \frac{a^2+6a+9}{a^2-2a+3} \leq 24$

Đẳng thức xảy ra khi $a=b=c>0$

 

 

 

Bài 63:

Cho ba số thực dương a,b,c. CMR $\sum \frac{a\sqrt{b}}{\sqrt{a}+2\sqrt{b}}\leq \frac{a+b+c}{3}$

 

Giải:

$\frac{a\sqrt{b}}{\sqrt{a}+2\sqrt{b}}\leq \frac{5a+b}{18}\Leftrightarrow \frac{(5\sqrt{a}+2\sqrt{b})(\sqrt{a}-\sqrt{b})^2}{18(\sqrt{a}+2\sqrt{b})}\geq 0$ ( đúng vì $a,b>0$ )

Tương tự $....$

$\Rightarrow \sum \frac{a\sqrt{b}}{\sqrt{a}+2\sqrt{b}} \leq \frac{6(a+b+c)}{18}=\frac{a+b+c}{3}$

Dấu $"="$$\Leftrightarrow a=b=c$

Cách khác:

Ta có:

$\sum \dfrac{a\sqrt{b}}{\sqrt{a}+2\sqrt{b}} \le \dfrac{a+b+c}{3}$

$\Longleftrightarrow \dfrac{2a\sqrt{b}}{\sqrt{a}+2\sqrt{b}}\le \dfrac{2(a+b+c)}{3}$

$\Longleftrightarrow \left (a-\dfrac{a\sqrt{a}}{2\sqrt{b}+\sqrt{a}} \right ) \le \dfrac{2(a+b+c)}{3}$

$\Longleftrightarrow \dfrac{a\sqrt{a}}{2\sqrt{b}+\sqrt{a}} \ge \dfrac{a+b+c}{3}$

Áp dụng bất đẳng thức Cauchy-Schwartz,ta có

$\sum \left (\dfrac{a\sqrt{a}}{2\sqrt{b}+\sqrt{a}} \right ) = \sum \left (\dfrac{a^2}{2\sqrt{ab}+a} \right ) \ge \dfrac{(a+b+c)^2}{(\sqrt{a}+\sqrt{b}+\sqrt{c})^2} \ge \dfrac{a+b+c}{3}$

Cách khác:

Sử dụng bất đẳng thức $\frac{9}{x+y+z}\leqslant \frac{1}{x}+\frac{1}{y}+\frac{1}{z}$ ta có

            $\frac{9}{\sqrt{a}+2\sqrt{b}}\leqslant \frac{1}{\sqrt{a}}+\frac{2}{\sqrt{b}}$

$\Rightarrow \frac{9a\sqrt{b}}{\sqrt{a}+2\sqrt{b}}\leqslant \frac{a\sqrt{b}}{\sqrt{a}}+\frac{2a\sqrt{b}}{\sqrt{b}}=\sqrt{ab}+2a$

Tương tự 2 bất đẳng thức còn lại rồi cộng vào ta được

           $\sum \frac{9a\sqrt{b}}{\sqrt{a}+2\sqrt{b}}\leqslant \sqrt{ab}+\sqrt{bc}+\sqrt{ac}+2(a+b+c)\leqslant 3(a+b+c)$

 $\sum \frac{a\sqrt{b}}{\sqrt{a}+2\sqrt{b}}\leqslant \frac{a+b+c}{3}$

Đẳng thức xảy ra khi $a=b=c>0$

 

 

 

Bài 64:

Cho $\left\{\begin{matrix} a,b,c\geq 0 & \\a+b+c=1 & \end{matrix}\right.$

 

CMR
$\sum \sqrt{a^{2}+2ab+2b^{2}}\geq \sqrt{5}$

Giải:

Đpcm$\Leftrightarrow \sum \sqrt{(a+b)^{2}+b^{2}}\geq \sqrt{5}$

Ta có $(a+b)^{2}+b^{2}=\frac{1}{5}[(a+b)^{2}+b^{2}](1^{2}+2^{2})\geq \frac{1}{5}[2(a+b)+b]^{2}=\frac{1}{5}(2a+3b)^{2}$

$\Rightarrow \sqrt{(a+b)^{2}+b^{2}}\geq \frac{1}{\sqrt{5}}(2a+3b)$

Hoàn toàn tương tự: $\sqrt{(b+c)^{2}+c^{2}}\geq \frac{1}{\sqrt{5}}(2b+3c)$

                                 $\sqrt{(c+a)^{2}+a^{2}}\geq \frac{1}{\sqrt{5}}(2c+3a)$

Cộng vế với vế của các BĐT cùng chiều ta có đpcm

Dấu bằng đạt $\Leftrightarrow a=b=c=\frac{1}{3}$

Cách khác:

$\sqrt{a^2+2ab+2b^2}\geq \frac{2a+3b}{\sqrt{5}}\Leftrightarrow (a-b)^2\geq 0$ ( luôn đúng $\forall a,b$ )

Tương tự ...

Suy ra $\sum \sqrt{a^{2}+2ab+2b^{2}}\geq \frac{5(a+b+c)}{\sqrt{5}}=\frac{5}{\sqrt{5}}=\sqrt{5}$

Dấu bằng xảy ra khi $a=b=c=\frac{1}{3}$

 

 

 

Bài 65:

Cho các số không âm a,b,c thỏa mãn $a+b+c=3$.Chứng minh rằng

$$\frac{a}{b^3+16}+\frac{b}{c^3+16}+\frac{c}{a^3+16}\geq\frac{1}{6}$$

Giải:

Ta có : $\sum \dfrac{a}{b^3+16}$=$\sum \dfrac{a}{16}-\sum \dfrac{ab^3}{16(b^3+16)}$ $\geq \dfrac{a+b+c}{16}-\dfrac{ab^2+bc^2+ca^2}{192}$

Vậy ta chỉ cần chứng minh:

$$ab^2+bc^2+ca^2 \leq 4$$

Đây là 1 bổ đề quen thuộc:

$$ab^2+bc^2+ca^2+abc\leq \dfrac{4(a+b+c)^3}{27}$$

Chứng minh: giả sử $b$ là số nằm giữa $a$ và $c$.

Suy ra: $a(a-b)(b-c)\geq 0$ $\Leftrightarrow abc+a^2b \geq ab^2+ca^2$

Ta có: $ab^2+bc^2+ca^2+abc \leq bc^2+2abc+a^2b=b(a+c)^2 \leq \dfrac{4(a+b+c)^3}{27}$

Bất đẳng thức được chứng minh.

Dấu đẳng thức xảy ra khi $(a,b,c)=(0,1,2)$ cùng các hoán vị.

 

 

 

Bài 66:

Cho a,b,c là số dương.Tìm min của P= $\frac{a+b}{a+b+c}+\frac{b+c}{4a+b+c}+\frac{c+a}{16b+c+a}$

Giải:

Đặt: $\left\{\begin{matrix} a+b+c=x & & \\ 4a+b+c=2y & & \\ 16b+c+a=6z& & \\ \end{matrix}\right.\Rightarrow \left\{\begin{matrix} a=\frac{2y-x}{3} & & \\ b=\frac{6z-x}{15} & & \\ c=\frac{21x-10y-6z}{15} & &\\ \end{matrix}\right.$

$\Rightarrow \left\{\begin{matrix} a+b=\frac{10y+6z-6x}{15} & & \\ b+c=\frac{4x-2y}{3} & & \\ c+a=\frac{16x-6z}{15} \end{matrix}\right.$

Khi đó ta có: $\left\{\begin{matrix} \frac{a+b}{a+b+c}=\frac{10y+6z-6x}{15x}=\frac{2y}{3x}+\frac{2z}{5x}-\frac{2}{5} & & \\ \frac{b+c}{4a+b+c}=\frac{4x-2y}{6y}=\frac{2x}{3y}-\frac{1}{3} & & \\ \frac{c+a}{16b+c+a}=\frac{16x-6z}{90z}=\frac{8x}{45z}-\frac{1}{15} & &\\ \end{matrix}\right.$

Từ đó, ta viết lại $P$ thành:

$P=\left ( \frac{2x}{3y}+\frac{2y}{3x} \right )+\left ( \frac{2z}{5x}+\frac{8x}{45z} \right )-\frac{4}{5}\geq \frac{4}{3}+\frac{8}{15}-\frac{4}{5}=\frac{16}{15}$

Vậy GTNN của $P$ là $\frac{16}{15}$

Đẳng thức xảy ra là bao gồm tất cả bộ số $(x, y, z)$ thỏa mãn hệ phương trình: $\left\{\begin{matrix} \frac{x}{y}=\frac{y}{x} & & \\ \frac{2z}{5x}=\frac{8x}{45z} & & \end{matrix}\right.$

 

 

Bài 67:

Cho 2 số thực khác nhau $a;b$. Chứng minh

 

$$\frac{(a-b)^4}{a^4+6a^2b^2+b^4}+\frac{4ab}{(a-b)^2} \geq 1$$

Giải:

Sử dụng hằng đẳng thức ta có 

BĐT $\Leftrightarrow \frac{a^4+b^4+6a^2b^2-4ab(a^2+b^2)}{a^4+6a^2b^2+b^4}+\frac{4ab}{(a-b)^2} \geqslant 1$

         $\Leftrightarrow \frac{ab}{(a-b)^2} \geqslant \frac{ab(a^2+b^2)}{a^4+b^4+6a^2b^2}$

         $\Leftrightarrow ab(a^4+b^4+6a^2b^2) \geqslant ab(a^2+b^2)(a-b)^2$

         $\Leftrightarrow ab\left [ a^4+b^4+6a^2b^2-(a-b)^2(a^2+b^2) \right ] \geqslant 0$

         $\Leftrightarrow 2a^2b^2.(2ab+a^2+b^2) \geqslant 0$

         $\Leftrightarrow 2a^2b^2(a+b)^2 \geqslant 0$

Bất đẳng thức trên luôn đúng với mọi $a,b$

Đẳng thức xảy ra khi $a=0,b=0$ hoặc $a+b=0$

 

 

 

Bài 68:

Cho a , b , c là ba số thực dương và a + b + c = 1 . Tìm GTLN của :
\[P = \sqrt {\frac{{ab}}{{\left( {1 - a} \right)\left( {1 - b} \right)}}}  + \sqrt {\frac{{bc}}{{\left( {1 - b} \right)\left( {1 - c} \right)}}}  + \sqrt {\frac{{ca}}{{\left( {1 - c} \right)\left( {1 - a} \right)}}} \]

Giải:

Áp dụng bất đẳng thức Cauchy ta có:
$P\leq \frac{1}{2}(\frac{a}{1-b}+\frac{b}{1-a}+\frac{b}{1-c}+\frac{c}{1-b}+\frac{c}{1-a}+\frac{a}{1-c})=\frac{1}{2}(\frac{1-a}{1-a}+\frac{1-b}{1-b}+\frac{1-c}{1-c})=\frac{3}{2}$.

Dấu bằng xảy ra khi a=b=c=1/3.

 

 

 

Bài 69:

Ch0 $a,b,c$ thỏa mãn $a,b,c \in \left [ 1;2 \right ]$

Chứng minh rằng $a^3+b^3+c^3 \leqslant 5abc$

Giải:

Viết lại bất đẳng thức cần chứng minh thành: $P=\frac{a^{2}}{bc}+\frac{b^{2}}{ca}+\frac{c^{2}}{ab}\leq 5$

 

Do vai trò bình đẳng của $a,b,c$ nên giả sử: $1\leq a\leq b\leq c\leq 2$. Khi đó: $(a-b)(b^{2}-c^{2})\geq 0$

 

                                     $\Leftrightarrow b^{3}\leq ab^{2}+bc^{2}-ac^{2}$

 

                                     $\Leftrightarrow \frac{b^{2}}{ca}\leq \frac{b}{c}+\frac{c}{a}-\frac{c}{b}$           

 

Mặt khác, ta có: $\frac{a^{2}}{bc}\leq \frac{a^{2}}{ac}=\frac{a}{c}$ và $\frac{c^{2}}{ab}\leq \frac{2c}{ab}=\frac{2c}{b}$

 

Cộng vế với vế ba bất đẳng thức trên ta có: $P\leq \left ( \frac{b}{c}+\frac{c}{b} \right )+\left ( \frac{a}{c}+\frac{c}{a} \right )$

 

Vì $b\leq c\leq 2\leq 2b$ nên $\frac{2b}{c}\geq 1, \frac{c}{b}\geq 1>\frac{1}{2}$

 

Do đó: $\left ( \frac{2b}{c}-1 \right )\left ( \frac{c}{b}-\frac{1}{2} \right )\geq 0\Rightarrow \frac{b}{c}+\frac{c}{b}\leq \frac{5}{2}$

 

Tương tự ta có: $\frac{a}{c}+\frac{c}{a}\leq \frac{5}{2}$

 

Do đó ta có: $P\leq 5$

 

Đẳng thức xảy ra khi và chỉ khi $a=b=1, c=2$ và các hoán vị

Cách khác:

Giả sử $a\geq b\geq c$

Dễ thấy $(a-2))(a^{2}+2a-1)\leq 0\Leftrightarrow a^{3}+2\leq 5a$

$(b-1)(b^{2}+b+1-5a)\leq 0\Leftrightarrow 5a+b^{3}\leq 5ab+1$

$(c-1)(c^{2}+c+1-5ab)\leq 0\Leftrightarrow 5ab+c^{3}\leq 5abc+1$

 (Vì $b^{2}+b+1\leq a^{2}+a+1\leq 5a$

 

 

 

Bài 70:

Cho a,b,c là các số dương và $ab+bc+ca=2abc$ Chứng minh rằng:

$\frac{1}{a(2a-1)^{2}}+\frac{1}{b(2b-1)^{2}}+\frac{1}{c(2c-1)^{2}}\geq \frac{1}{2}$

Giải:

ĐK : $a,b,c\neq \frac{1}{2}$ . Chia cả 2 vế của giả thiết cho $ abc \neq 0 $ ta được $\frac{1}{a}+\frac{1}{b}+\frac{1}{c}=2$

$\frac{1}{a(2a-1)^2}\geq \frac{1}{a}-\frac{1}{2}$

$\Leftrightarrow \frac{2+(a-2)(2a-1)^2}{2a(2a-1)^2}\geq 0$ $\Leftrightarrow \frac{a(2a-3)^2}{2a(2a-1)^2}\geq 0$ Điều này đúng vì $a\neq \frac{1}{2}$ ; $a> 0$

Dấu bằng xảy ra khi $a=b=c=\frac{3}{2}$

 

 

 

Bài 71:

Cho $a,b,c$ dương và $a+b+c=2013$ Tìm GTLN của $T=\frac{(2013-a)(2013-b)(2013-c)}{(2013+a)(2013+b)(2013+c)}$

Giải:

$T=\frac{(a+b)(b+c)(c+a)}{(2a+b+c)(2b+c+a)(2c+a+b)}$

Đặt $x=a+b,y=b+c,z=c+a$

Ta có $T=\frac{xyz}{(x+y)(y+z)(z+x)}\leq \frac{1}{8}$

Dấu "$=$" xảy ra $\Leftrightarrow a=b=c=671$

 

 

Bài 72:

Cho $a,b,c\geq 0$ thoả mãn $a^{2}+b^{2}+c^{2}\leq 3b$.

Tìm GTNN: P= $\frac{1}{(a+1)^{2}}+\frac{4}{(b+2)^{2}}+\frac{8}{(c+3)^{2}}$

Giải:

Áp dụng BĐT $\frac{1}{t^2}+\frac{1}{p^2}\geq \frac{8}{(t+p)^2}$, ta có $P=\frac{1}{(a+1)^2}+\frac{1}{(\frac{b}{2}+1)^2}+\frac{8}{(c+3)^2}\geq \frac{8}{(a+\frac{b}{2}+2)^2}+\frac{8}{(c+3)^2}\geq \frac{64}{(a+\frac{b}{2}+c+5)^2}$

(1)

Từ giả thiết, ta có $3b+6\geq (a^2+1)+(b^2+4)+(c^2+1)\geq 2a+4b+2c\Rightarrow 6\geq 2a+b+2c\Rightarrow 3\geq a+\frac{b}{2}+c$             

(2)

Từ (1) và (2) suy ra GTNN của P là 8

 

 

 

Bài 73:

Cho x,y,z là các số thực dương thỏa mãn xyz=1.chứng minh rằng :

$\frac{x+y}{x+y+1}+\frac{y+z}{y+z+1}+\frac{z+x}{z+x+1}\geq 2$

Giải:

Ta có: $\frac{x+y}{x+y+1}=1-\frac{1}{x+y+1}$

 

Do đó bất đẳng thức cần chứng minh tương đương với: 

 

$\frac{1}{x+y+1}+\frac{1}{y+z+1}+\frac{1}{z+x+1}\leq 1$

 

Đặt $x=a^{3},y=b^{3},z=c^{3}$, ta có $abc=1$ và bất đẳng thức trên tương đương:

 

$\frac{1}{1+a^{3}+b^{3}}+\frac{1}{1+b^{3}+c^{3}}+\frac{1}{1+c^{3}+a^{3}}\leq 1$

 

Ta có: $a^{3}+b^{3}=(a+b)(a^{2}-ab+b^{2})\geq ab(a+b)$

 

Do đó: $a^{3}+b^{3}+1\geq ab(a+b)+abc=ab(a+b+c)$ nên $\frac{1}{1+a^{3}+b^{3}}\leq \frac{c}{a+b+c}$

 

Thiết lập hai bất đẳng thức tương tự rồi cộng vế với vế ta có đpcm

 

 

Bài 74:

Cho a,b,c,d là các số thực dương và abcd=1. Tìm Max P=$\frac{1}{3+a}+\frac{1}{3+b}+\frac{1}{3+c}+\frac{1}{3+d}$

Giải:

Ta xét hàm $f(x)=\frac{1}{3+x}+\dfrac{1}{16} \ln x-\frac{1}{4}$
$f'(x)=0$ khi và chỉ khi $x=1$ hoặc $x=9$
Nếu $0<x < 9$ thì $f(x) \leq f(1)=0$
Suy ra $\frac{1}{3+x}+\dfrac{1}{16} \ln x-\frac{1}{4} \leq 0$
_________________
Tóm lại: Nếu $a,b,c,d$ có 1 số $>9$ (giả sử là $d>9$) thì $P \leq \frac{3}{4}-\dfrac{1}{16} \ln \frac{1}{d}+\frac{1}{3+d}<1$
Nếu $a,b,c,d$ có 2 số $>9$ (giả sử là $c,d>9$) thì $P \leq \frac{1}{2}-\dfrac{1}{16} \ln \frac{1}{cd}+\frac{1}{3+c}+\frac{1}{3+d}<1$
Nếu $a,b,c,d$ có 3 số $>9$ (giả sử là $b,c,d>9$) thì luôn có $P<1$
Nếu $a,b,c,d$ có 4 số $>9$ thì luôn cũng có $P<1$
Nếu $a,b,c,d$ đều $<9$ thì theo bổ đề trên $P \leq 1-\frac{1}{16} \ln 1=1$
Suy ra OK

 

 

 

Bài 75:

Cho $a, b, c >0$ và $a+b+c=3$ tìm max

 

$\frac{2}{3+ab+bc+ca}+\sqrt[3]{\frac{abc}{(1+a)(1+b)(1+c)}}$

 

Giải:

Từ giả thiết suy ra:

$$abc\leq \left ( \frac{a+b+c}{3} \right )^3=1$$

Lại có:

$$\left ( ab+bc+ca \right )^2\geq 3abc\left ( a+b+c \right )=9abc$$

$$\Rightarrow ab+bc+ca\geq 3\sqrt{abc}$$

Ta có:

$$P=\frac{2}{3+ab+bc+ca}+\sqrt[3]{\frac{abc}{\left ( 1+a \right )\left ( 1+b \right )\left ( 1+c \right )}}$$

$$=\frac{2}{3+ab+bc+ca}+\sqrt[3]{\frac{abc}{4+ab+bc+ca+abc}}$$

$$\leq \frac{2}{3+3\sqrt{abc}}+\sqrt[3]{\frac{abc}{4+3\sqrt{abc}+abc}}$$

Đặt:

$$f\left ( x \right )=\frac{2}{3+3\sqrt{x}}+\sqrt[3]{\frac{x}{4+3\sqrt{x}+x}},(x=abc,0<x \le 1)$$

Khi đó, ta có:

$f'(x)>0$

$\Rightarrow$ Hàm $f$ đồng biến trong $(0;1]$

Do đó:

$max P=max f(x)=\frac{5}{6}$

Đẳng thưc xảy ra $\Leftrightarrow x=1 \Leftrightarrow a=b=c=1$

Cách khác:

$\frac{2}{3+ab+bc+ca}+\sqrt[3]{\frac{abc}{(1+a)(1+b)(1+c)}}$
$=\frac{2}{9}.\frac{(a+b+c)^2}{(a+1)b+(b+1)c+(c+1)a}+\sqrt[3]{\frac{abc}{(1+a)(1+b)(1+c)}}$

$\leq\frac29.\sum\frac{1}{1+a}+\frac39\sum\frac{a}{1+a}=\frac23+\frac19.\sum\frac{a}{1+a}$

$\leq\frac23+\frac{1}{18}.\sum\sqrt{a}\leq\frac23+\frac{1}{18}.\sqrt{3(a+b+c)}=\frac56$

 

 

 

Bài 76:

Cho $a,b,c \geq 0$ thoả mãn $a^2+b^2+c^2=3$

Tìm Min của $P=\frac{(a+b+c-1)^2}{a^2b+b^2c+c^2a}+\frac{1}{a}+\frac{1}{b}+\frac{1}{c}$

Giải:

Ta có $(a^2+b^2+c^2)(a+b+c)\geq 3(a^2b+b^2c+c^2a)$

$\Rightarrow a+b+c\geq a^2b+b^2c+c^2a$ (vì $a^2+b^2+c^2=3$)

$P\geq \frac{(a+b+c-1)^2}{a+b+c}+\frac{9}{a+b+c}=\sum a+\frac{9}{\sum a}+\frac{1}{\sum a}-2$

Ta lại có $\sum a\leq \sqrt{3\sum a^2}=3$

Suy ra $P_{Min}=\frac{13}{3}$ dấu $"="$ $\Leftrightarrow a=b=c=1$

 

 

 

Bài 77:

Cho $ a,b,c>0$ thỏa mãn $a+b+c=1$. Chứng minh rằng:

$$\frac{a^2}{9a+1}+\frac{b^2}{9b+1}+\frac{c^2}{9c+1} \leq \frac{1}{12\sqrt{3(ab+bc+ca)}}$$

Giải:

Ta có: $\frac{a^{2}}{9a+1}=\frac{a}{9}-\frac{a}{9(9a+1)}$ nên bất đẳng thức cần chứng minh tương đương với:

 

    $\frac{a}{9a+1}+\frac{b}{9b+1}+\frac{c}{9c+1}+\frac{3}{4\sqrt{3(ab+bc+ca)}}\geq 1$

 

Áp dụng bất đẳng thức Cauchy - Schwarz và AM-GM ta có:

 

   $\frac{a}{9a+1}+\frac{b}{9b+1}+\frac{c}{9c+1}\geq \frac{(a+b+c)^{2}}{a(9a+1)+b(9b+1)+c(9c+1)}=\frac{1}{9(a^{2}+b^{2}+c^{2})+1}$

 

  $\frac{3}{4\sqrt{3(ab+bc+ca)}}\geq \frac{3}{6(ab+bc+ca)+2}$

 

Áp dụng bất đẳng thức Cauchy - Schwarz ta có:

 

  $\frac{1}{9(a^{2}+b^{2}+c^{2})+1}+\frac{3}{6(ab+bc+ca)+2}\geq \frac{(1+3)^{2}}{9(a+b+c)^{2}+7}=1$

 

Do đó ta có điều phải chứng minh.

 

 

Bài 78:

Cho $x,y,z$ thực không âm thỏa $z=\max \{x,y,z\}$ và $xy+xz+yz>0$. Tìm min của biêu thức $$P=\frac{x}{y+z}+2\sqrt{\frac{y}{x+z}}+3\sqrt[3]{\frac{z}{x+y}}$$

 

Giải:

 Để ý với điều kiện $x,y,z$ không âm, ta nghĩ ngay đến chuyện dấu bằng xảy ra khi có 1 số bằng 0. Lần mò thêm tý nữa, ta có thể dự đoán dấu bằng xảy ra khi $y=0,z=x$ và $P=4$.

 

Lại nhìn vào biểu thức $P$ và điều kiện $z=max\{x;y;z\}$, thật tự nhiên ta nghĩ đến việc đưa biểu thức về 1 biến để khảo sát hàm số, cụ thể là $\sqrt[3]{\frac{z}{x+y}}$. Số hạng này mang tính đối xứng giữa 2 biến $x,y$ nên ta cũng sẽ đi tìm cách đánh giá đưa $P$ về đối xứng the0 $x,y$ bằng việc sử dụng 1 số bất đẳng thức quen thuộc :)

Áp dụng liên tiếp AM-GM ta có :

$$\frac{x}{y+z}+2\sqrt{\frac{y}{x+z}}\geq 2\left(\sqrt{\frac{x}{y+z}}+\sqrt{\frac{y}{x+z}}\right)-1$$

$$=4\left(\frac{x}{2\sqrt{x(y+z)}}+\frac{y}{2\sqrt{y(x+z)}}\right)-1$$

$$\geq \frac{4(x+y)}{x+y+z}-1=\dfrac{4}{1+\frac{z}{x+y}}-1$$

Giờ thì đặt $\frac{z}{x+y}=t^3$ với $t\geq \sqrt[3]{\frac{1}{2}}$ ta có:

$$P\geq \frac{4}{1+t^3}+3t-1=f(t)$$

Ta có $f'(t)=4.\frac{(t-1)(t^2+t-1)(t^3+2t+1)}{(t^3+1)^2}$  nhưng do $t^2+t-1>0$ (Do $t\geq \sqrt[3]{\frac{1}{2}}$) nên $f(t)$ đạt min khi $t=1$ và $P=4$

Kết thúc chứng minh, đẳng thức xảy ra tại $x=z,y=0$ $\blacksquare$

 

 

 

Bài 79:

Tìm minP theo 2 cách:

 

$P=\frac{1}{2+4a}+\frac{1}{3+9b}+\frac{1}{6+36c}$

 

trong đó a,b,c là 3 số thực dương thoả mãn $a+b+c=1$

Giải:

Áp dụng bất đẳng thức $Cauchy,$ ta có:

$$\frac{1}{2+4a}+\frac{2+4a}{16}\geq \frac{1}{2}$$

$$\frac{1}{3+9a}+\frac{3+9a}{36}\geq \frac{1}{3}$$

$$\frac{1}{6+36a}+\frac{6+36a}{144}\geq \frac{1}{6}$$

Do đó: $$\frac{1}{2+4a}+\frac{1}{3+9b}+\frac{1}{6+36c}+\frac{2+4a}{16}+\frac{3+9a}{36}+\frac{6+36a}{144}\geq 1$$

$$\Leftrightarrow P=\frac{1}{2+4a}+\frac{1}{3+9b}+\frac{1}{6+36c}\geq 1-\left (\frac{2+4a}{16}+\frac{3+9a}{36}+\frac{6+36a}{144} \right )$$

$$\Leftrightarrow P=\frac{1}{2+4a}+\frac{1}{3+9b}+\frac{1}{6+36c}\geq 1-\frac{1}{4}-\frac{a+b+c}{4}=\frac{1}{2}$$

Vậy giá trị nhỏ nhất của $P$ là $\frac{1}{2}$ khi $a=\frac{1}{2};\ b=\frac{1}{3};\ c=\frac{1}{6}$

 

 

 

Bài 80:

Cho các số dương $a,b,c$ thoả mãn $ab^2+bc^2+ca^2=3$ .Chứng minh rằng:

$\sum \frac{2a^5+3b^5}{ab} \geq 15(a^3+b^3+c^3-2)$

Giải:

Bài này chỉ phức tạp ở công thức biến đổi: Bất đẳng thức cần chứng minh tương đương:

$\sum \frac{(a-b)^{4}(2a+3b)}{ab}\geq 0$ và tất nhiên là đúng (Bạn có thể tìm cách biến đổi hoặc nếu chưa chắc thì nhân tung biểu thức đó ra sẽ ra biểu thức ban đầu, mình thử rồi)

Trang 18

 

 

 

 

 

 

Bài 3 của bạn có vấn đề ở dấu bằng. Dấu bằng không xảy ra ở đó






3 người đang xem chủ đề

0 thành viên, 3 khách, 0 thành viên ẩn danh